Difference between revisions of "2022 AMC 10A Problems/Problem 18"

(Redirected page to 2022 AMC 12A Problems/Problem 18)
(Tag: New redirect)
 
(Solution 2)
 
(18 intermediate revisions by 6 users not shown)
Line 1: Line 1:
#redirect [[2022 AMC 12A Problems/Problem 18]]
+
{{duplicate|[[2022 AMC 10A Problems/Problem 18|2022 AMC 10A #18]] and [[2022 AMC 12A Problems/Problem 18|2022 AMC 12A #18]]}}
 +
 
 +
==Problem==
 +
 
 +
Let <math>T_k</math> be the transformation of the coordinate plane that first rotates the plane <math>k</math> degrees counterclockwise around the origin and then reflects the plane across the <math>y</math>-axis. What is the least positive
 +
integer <math>n</math> such that performing the sequence of transformations <math>T_1, T_2, T_3, \cdots, T_n</math> returns the point <math>(1,0)</math> back to itself?
 +
 
 +
<math>\textbf{(A) } 359 \qquad \textbf{(B) } 360 \qquad \textbf{(C) } 719 \qquad \textbf{(D) } 720 \qquad \textbf{(E) } 721 </math>
 +
 
 +
==Solution 1==
 +
Let <math>P=(r,\theta)</math> be a point in polar coordinates, where <math>\theta</math> is in degrees.
 +
 
 +
Rotating <math>P</math> by <math>k^{\circ}</math> counterclockwise around the origin gives the transformation <math>(r,\theta)\rightarrow(r,\theta+k^{\circ}).</math> Reflecting <math>P</math> across the <math>y</math>-axis gives the transformation <math>(r,\theta)\rightarrow(r,180^{\circ}-\theta).</math> Note that
 +
<cmath>\begin{align*}
 +
T_k(P)&=(r,180^{\circ}-\theta-k^{\circ}), \\
 +
T_{k+1}(T_k(P)) &= (r,\theta -1^{\circ}).
 +
\end{align*}</cmath>
 +
We start with <math>(1,0^{\circ})</math> in polar coordinates. For the sequence of transformations <math>T_1, T_2, T_3, \cdots, T_k,</math> it follows that
 +
 
 +
* After <math>T_1,</math> we have <math>(1,179^{\circ}).</math>
 +
 
 +
* After <math>T_2,</math> we have <math>(1,-1^{\circ}).</math>
 +
 
 +
* After <math>T_3,</math> we have <math>(1,178^{\circ}).</math>
 +
 
 +
* After <math>T_4,</math> we have <math>(1,-2^{\circ}).</math>
 +
 
 +
* After <math>T_5,</math> we have <math>(1,177^{\circ}).</math>
 +
 
 +
* After <math>T_6,</math> we have <math>(1,-3^{\circ}).</math>
 +
 
 +
* ...
 +
 
 +
* After <math>T_{2k-1},</math> we have <math>(1,180^{\circ}-k^{\circ}).</math>
 +
 
 +
* After <math>T_{2k},</math> we have <math>(1,-k^{\circ}).</math>
 +
 
 +
The least such positive integer <math>k</math> is <math>180.</math> Therefore, the least such positive integer <math>n</math> is <math>2k-1=\boxed{\textbf{(A) } 359}.</math>
 +
 
 +
~MRENTHUSIASM
 +
 
 +
==Solution 2==
 +
Note that since we're reflecting across the <math>y</math>-axis, if the point ever makes it to <math>(-1,0)</math> then it will flip back to the original point. Note that after <math>T_1</math> the point will be <math>1</math> degree clockwise from the negative <math>x</math>-axis. Applying <math>T_2</math> will rotate it to be <math>1</math> degree counterclockwise from the negative <math>x</math>-axis, and then flip it so that it is <math>1</math> degree clockwise from the positive <math>x</math>-axis. Therefore, after every <math>2</math> transformations, the point rotates <math>1</math> degree clockwise. To rotate it so that it will rotate <math>179</math> degrees clockwise will require <math>179 \cdot 2 = 358</math> transformations. Then finally on the last transformation, it will rotate on to <math>(-1,0)</math> and then flip back to its original position. Therefore, the answer is <math>358+1 = 359 = \boxed{\textbf{(A) } 359}</math>.
 +
 
 +
~KingRavi
 +
 
 +
==Solution 3==
 +
In degrees:
 +
 
 +
Starting with <math>n=0</math>, the sequence goes <math>{0}\rightarrow {179}\rightarrow {359}\rightarrow {178}\rightarrow {358}\rightarrow {177}\rightarrow {357}\rightarrow\cdots.</math>
 +
 
 +
We see that it takes <math>2</math> steps to downgrade the point by <math>1^{\circ}</math>. Since the <math>1</math>st point in the sequence is <math>{179}</math>, the answer is <math>1+2(179)=\boxed{\textbf{(A) } 359}.</math>
 +
 
 +
==Video Solution==
 +
 
 +
https://youtu.be/QQrsKTErJn8
 +
 
 +
~Steven Chen (Professor Chen Education Palace, www.professorchenedu.com)
 +
 
 +
==Video Solution (Simple and Fun!!!)==
 +
https://youtu.be/7yAh4MtJ8a8?si=2UC_9X7DjkL8UW5C&t=4968
 +
 
 +
~Math-X
 +
 
 +
==See also==
 +
{{AMC10 box|year=2022|ab=A|num-b=17|num-a=19}}
 +
{{AMC12 box|year=2022|ab=A|num-b=17|num-a=19}}
 +
{{MAA Notice}}

Latest revision as of 16:12, 1 November 2023

The following problem is from both the 2022 AMC 10A #18 and 2022 AMC 12A #18, so both problems redirect to this page.

Problem

Let $T_k$ be the transformation of the coordinate plane that first rotates the plane $k$ degrees counterclockwise around the origin and then reflects the plane across the $y$-axis. What is the least positive integer $n$ such that performing the sequence of transformations $T_1, T_2, T_3, \cdots, T_n$ returns the point $(1,0)$ back to itself?

$\textbf{(A) } 359 \qquad \textbf{(B) } 360 \qquad \textbf{(C) } 719 \qquad \textbf{(D) } 720 \qquad \textbf{(E) } 721$

Solution 1

Let $P=(r,\theta)$ be a point in polar coordinates, where $\theta$ is in degrees.

Rotating $P$ by $k^{\circ}$ counterclockwise around the origin gives the transformation $(r,\theta)\rightarrow(r,\theta+k^{\circ}).$ Reflecting $P$ across the $y$-axis gives the transformation $(r,\theta)\rightarrow(r,180^{\circ}-\theta).$ Note that \begin{align*} T_k(P)&=(r,180^{\circ}-\theta-k^{\circ}), \\ T_{k+1}(T_k(P)) &= (r,\theta -1^{\circ}). \end{align*} We start with $(1,0^{\circ})$ in polar coordinates. For the sequence of transformations $T_1, T_2, T_3, \cdots, T_k,$ it follows that

  • After $T_1,$ we have $(1,179^{\circ}).$
  • After $T_2,$ we have $(1,-1^{\circ}).$
  • After $T_3,$ we have $(1,178^{\circ}).$
  • After $T_4,$ we have $(1,-2^{\circ}).$
  • After $T_5,$ we have $(1,177^{\circ}).$
  • After $T_6,$ we have $(1,-3^{\circ}).$
  • ...
  • After $T_{2k-1},$ we have $(1,180^{\circ}-k^{\circ}).$
  • After $T_{2k},$ we have $(1,-k^{\circ}).$

The least such positive integer $k$ is $180.$ Therefore, the least such positive integer $n$ is $2k-1=\boxed{\textbf{(A) } 359}.$

~MRENTHUSIASM

Solution 2

Note that since we're reflecting across the $y$-axis, if the point ever makes it to $(-1,0)$ then it will flip back to the original point. Note that after $T_1$ the point will be $1$ degree clockwise from the negative $x$-axis. Applying $T_2$ will rotate it to be $1$ degree counterclockwise from the negative $x$-axis, and then flip it so that it is $1$ degree clockwise from the positive $x$-axis. Therefore, after every $2$ transformations, the point rotates $1$ degree clockwise. To rotate it so that it will rotate $179$ degrees clockwise will require $179 \cdot 2 = 358$ transformations. Then finally on the last transformation, it will rotate on to $(-1,0)$ and then flip back to its original position. Therefore, the answer is $358+1 = 359 = \boxed{\textbf{(A) } 359}$.

~KingRavi

Solution 3

In degrees:

Starting with $n=0$, the sequence goes ${0}\rightarrow {179}\rightarrow {359}\rightarrow {178}\rightarrow {358}\rightarrow {177}\rightarrow {357}\rightarrow\cdots.$

We see that it takes $2$ steps to downgrade the point by $1^{\circ}$. Since the $1$st point in the sequence is ${179}$, the answer is $1+2(179)=\boxed{\textbf{(A) } 359}.$

Video Solution

https://youtu.be/QQrsKTErJn8

~Steven Chen (Professor Chen Education Palace, www.professorchenedu.com)

Video Solution (Simple and Fun!!!)

https://youtu.be/7yAh4MtJ8a8?si=2UC_9X7DjkL8UW5C&t=4968

~Math-X

See also

2022 AMC 10A (ProblemsAnswer KeyResources)
Preceded by
Problem 17
Followed by
Problem 19
1 2 3 4 5 6 7 8 9 10 11 12 13 14 15 16 17 18 19 20 21 22 23 24 25
All AMC 10 Problems and Solutions
2022 AMC 12A (ProblemsAnswer KeyResources)
Preceded by
Problem 17
Followed by
Problem 19
1 2 3 4 5 6 7 8 9 10 11 12 13 14 15 16 17 18 19 20 21 22 23 24 25
All AMC 12 Problems and Solutions

The problems on this page are copyrighted by the Mathematical Association of America's American Mathematics Competitions. AMC logo.png